LSAT 3 – Section 4 – Question 17

You need a full course to see this video. Enroll now and get started in less than a minute.

Target time: 0:58

This is question data from the 7Sage LSAT Scorer. You can score your LSATs, track your results, and analyze your performance with pretty charts and vital statistics - all with a Free Account ← sign up in less than 10 seconds

Question
QuickView
Type Tags Answer
Choices
Curve Question
Difficulty
Psg/Game/S
Difficulty
Explanation
PT3 S4 Q17
+LR
Sufficient assumption +SA
A
1%
144
B
1%
152
C
6%
154
D
75%
164
E
18%
157
140
150
161
+Medium 146.87 +SubsectionMedium
This page shows a recording of a live class. We're working hard to create our standard, concise explanation videos for the questions in this PrepTest. Thank you for your patience!

This is a sufficient assumption question because the question stem says: “conclusion follows logically…if which one… is assumed?”

Sufficient assumption questions tend to be very formal. We’re looking for a rule that would 100% validate the conclusion, specifically by bridging the premise and conclusion through the rule. Not only are we extrapolating the rule from our argument, but we’re also using that rule to render the argument “valid.” The way to prephrase our answer choice is by tying our premises and conclusion together into a rule: “If [premise] → then [conclusion].”

Our first sentence is very straightforward: in bureaucracies, decisions involve many people. The sentence just elaborates on this by saying that no one person have more authority than the next person. These are both our premises.

Our conclusion states that in bureaucracies, risky projects are never undertaken. That’s a big jump! From “bureaucracy decisions involving multiple people” to concluding “risky projects are never undertaken”? What if that is the exact they take on risky projects? For example, they could say that because many people are involved, they’re able to prepare for every outcome.

In order for our conclusion to follow, we need to link up the idea in the premises to the conclusion: when multiple people are involved in decisions and no one has the authority, risky projects will not be taken.

Answer Choice (A) This isn’t correct. We’re trying to make our conclusion about risky projects not being taken on valid; the fact that projects always require risk doesn’t help the gap in our argument.

Answer Choice (B) This isn’t correct either. The gap here is that the argument assumes that have many people involved in a decision means that no risky project will be taken. If we plug this into our stimulus, this doesn’t help validate our conclusion. It could support it, but there are too many assumption we need to make to arrive to our conclusion.

Answer Choice (C) This is more or less what our premises are trying to say, but again, it’s not helping to validate our conclusion about risky project not being taken on in bureaucracies. Additionally, we’re not concerned with what groups of people will take risks - we specifically interested in bureaucracies who will not take risks.

Correct Answer Choice (D) We said we weren’t interested in people who take risks, but this is a conditional statement! The answer choice is saying “when risk take, then single individual power to decide.” Taking the contrapositive of this would be: if multiple people have the power to decide, the risk is not taken. See how “no risk taken” is in the necessary condition? This is the NC in our rule, and it’s also our conclusion. Our premises trigger the contrapositive of this answer and allow us to draw our conclusion.

Answer Choice (E) This isn’t correct; what people do on there own is outside the scope of their decision as a group. This doesn’t help us draw our conclusion.

Take PrepTest

Review Results

Leave a Reply